LSAT and Law School Admissions Forum

Get expert LSAT preparation and law school admissions advice from PowerScore Test Preparation.

 Administrator
PowerScore Staff
  • PowerScore Staff
  • Posts: 8917
  • Joined: Feb 02, 2011
|
#81495
Complete Question Explanation

Assumption. The correct answer choice is (B).

Answer choice (A):

Answer choice (B): This is the correct answer choice.

Answer choice (C):

Answer choice (D):

Answer choice (E):

This explanation is still in progress. Please post any questions below!
 nmgee
  • Posts: 15
  • Joined: Jun 01, 2018
|
#46459
Hi,

I selected the correct answer to this problem (B), but did so by process of elimination because I did not see a stronger answer than B. Could you please explain why B is correct? I'm having trouble understanding why the "more likely" in answer choice B is a necessary assumption to the conclusion that the "syndrome is, in fact, caused by one virus". If it is only "more likely" to combat one virus than multiple, why is it considered evidence that the syndrome is indeed caused by one virus?
 Alex Bodaken
PowerScore Staff
  • PowerScore Staff
  • Posts: 136
  • Joined: Feb 21, 2018
|
#46562
nmgee,

Thanks for the question. Let me see if I can help. This is a question where the Assumption Negation Technique may be useful - as a reminder, this is where we logically negate an answer choice and see if it weakens the conclusion; if it does, it is the correct answer.

Answer choice (B) reads: "It is more likely that the new drug counteracts one virus than that it counteracts several viruses." The logical negation of this would read "It is NOT more likely that the new drug counteracts one virus than that it counteracts several viruses." Okay, so does this weaken the conclusion? It does, and here's how: if it isn't more likely that the new drug counteracts one virus as opposed to several, than we have no way of knowing if all of the symptoms (fatigue, pain, neurological symptoms) have been reduced because the drug is reacting to one virus that causes all of those symptoms or to multiple viruses causing these symptoms. This would weaken the conclusion. If, by contrast, it is more likely that the new drug counteracts one virus than that it counteracts several, then we have good evidence to conclude that the symptoms are being caused by one virus, since the virus is lessening all of them and yet is likely only responding to one virus.

Hope that helps!
Alex
 whardy21
  • Posts: 48
  • Joined: Sep 30, 2018
|
#66452
I initially chose B and then switched to D . Can you explain why D is incorrect.
User avatar
 KelseyWoods
PowerScore Staff
  • PowerScore Staff
  • Posts: 1079
  • Joined: Jun 26, 2013
|
#66819
Hi Whardy!

Trying out the Assumption Negation Technique can really help when you're trying to understand why answer choices are incorrect in Assumption questions. So let's try negating (taking the logical opposite) of answer choice (D) and see if/how it affects our argument.

To negate (D), let's just add a "not" so it becomes:

"Most syndromes that are characterized by related symptoms are NOT each caused by a single viral infection."

Does that negated statement attack our stimulus argument? Not quite. Because even if most syndromes are caused by multiple viruses rather than a single one, that doesn't mean that chronic fatigue syndrome isn't one of those rarer syndromes that is caused by only one virus.

Basically, we are concerned specifically about chronic fatigue syndrome. Information about these other syndromes is not necessary to the argument that there is evidence that chronic fatigue syndrome is caused only by one virus.

Hope this helps!

Best,
Kelsey
 Coleman
  • Posts: 44
  • Joined: Jul 07, 2020
|
#77222
Hi,

I understand the logic behind answer choice B which makes it a necessary assumption for the conclusion. (B) says "it is more likely that the new drug counteracts one virus than that it counteracts several viruses."
However, the phrase "more likely" doesn't seem appropriate in this context since it merely indicates correlation instead of causation. If we take it as a probability, for example, there is a 51 percent probability that the new drug counteracts one virus and 49 percent probability that it counteracts several viruses, therefore, it is "more likely" that the drug counteracts a single virus, there is no reason we should overlook the latter case.
If B was phrased more definite way, such as "the new drug counteracts only a single virus rather than several." I would definitely go with B without hesitation, but this "more likely" sounds pretty untempting. Could you give me some advice to interpret B?

Thanks in advance!
 Luke Haqq
PowerScore Staff
  • PowerScore Staff
  • Posts: 739
  • Joined: Apr 26, 2012
|
#77373
Hi Coleman!

I can certainly discuss answer choice (B).

First, this is an assumption question. Second, that answer states, "It is more likely that the new drug counteracts one virus than that it counteracts several viruses."

One strategy that is important to remember is to diagram conditional reasoning when you see it occur in a stimulus. There is a very good chance that such conditional reasoning will be necessary for arriving at the right answer, either by looking to the diagrammed statements themselves, or by figuring out new inferences that can be gleaned from them. In this question, we are given the following reasoning:

P1: newly synthesized drug :arrow: lessens symptoms
:longline:
C: evidence that it is caused by one virus rather than many

There's obviously a premise missing in this reasoning, which makes sense given the type of question. However, suppose we just created an obvious link with a couple more premises:

P1: newly synthesized drug :arrow: lessens symptoms (phenomenon)
(P2): lessens symptoms :arrow: lessens by acting on one virus rather than many (drug mechanism)
(P3): lessens by acting on one virus rather than many :arrow: evidence that it is caused by one virus rather than many (connecting drug mechanism to evidence about etiology)
:longline:
C: evidence that is caused by one virus rather than many (evidence about etiology)

In other words, the added premises are the assumption that the mechanism of this newly synthesized drug is that it acts on one rather than many viruses, and that the drug's efficacy provides evidence of the etiology of chronic fatigue syndrome. If these were assumed, they would enable one to get from P1 to the conclusion.

Further, these added premises are reflected in answer choice (B): "It is more likely that the new drug counteracts one virus than that it counteracts several viruses." This is most clearly getting at (P2), by noting an assumption being made about the drug mechanism of the newly synthesized drug. It also gets to (P3), by connecting drug's mechanism as an evidence about the syndrome. The "more likely" language is in line with the "there is evidence" language in the stimulus--if there is "more evidence" that something is the case, this is another way of saying that it is "more likely" that such a thing is the case.

Lastly, the Assumption Negation technique is also a valuable strategy to employ when considering answer choices on assumption questions. That strategy involves negating the answer choice and seeing if the conclusion still follows, or if the argument falls apart. Here, the negation of (B) would be "It is no more likely that the new drug counteracts one virus than that it counteracts several viruses." If this were true, it would seem impossible for the author to get from the drug's efficacy to the conclusion "Thus there is evidence that chronic fatigue syndrome is, in fact, caused by one virus."

Get the most out of your LSAT Prep Plus subscription.

Analyze and track your performance with our Testing and Analytics Package.